Gå til innhold
Trenger du skole- eller leksehjelp? Still spørsmål her ×

Den enorme matteassistansetråden


Anbefalte innlegg

Hei trenger hjelp med litt forståelse av likning

 

Jeg vet at dersom man skal dele eller gange i en likning, må det gjøres med hvert eneste ledd i likningen (der det skilles med + eller -)

 

Her vil det da være naturlig å gange med 2 og dele med m forran hvert eneste ledd

post-435273-0-73682700-1547903540_thumb.jpg

 

 

Da sitter man igjen med

post-435273-0-29281600-1547903829.jpg

 

Så kommer forvirringen

post-435273-0-95816200-1547903894.jpg

 

Jeg trodde man skulle ta rota over hvert ledd i likninger

f.eks= 100=25+3*25

Da trodde jeg rota av 100 ble selvsagt rota av 25 + rota av 3*25 men dette er feil

 

 

Poenget mitt var at jeg ikke kunne se hvordan bilde (3) ble til bilde (4)

tror jeg sovna når læreren gikk over rot i likning

Endret av Steeffen
Lenke til kommentar
Videoannonse
Annonse

Hei trenger hjelp med litt forståelse av likning

 

Jeg vet at dersom man skal dele eller gange i en likning, må det gjøres med hvert eneste ledd i likningen (der det skilles med + eller -)

 

Her vil det da være naturlig å gange med 2 og dele med m forran hvert eneste ledd

attachicon.gift1.JPG

 

 

Da sitter man igjen med

attachicon.gift2.JPG

 

Så kommer forvirringen

attachicon.gift3.JPG

 

Jeg trodde man skulle ta rota over hvert ledd i likninger

f.eks= 100=25+3*25

Da trodde jeg rota av 100 ble selvsagt rota av 25 + rota av 3*25 men dette er feil

 

 

Poenget mitt var at jeg ikke kunne se hvordan bilde (3) ble til bilde (4)

tror jeg sovna når læreren gikk over rot i likning

Det går ikke å komme direkte fra (3) til (4)

Det du kan skrive fra (3) er:

chart?cht=tx&chl=u_A = \sqrt{v_A^2 + 3 v_B^2}

 

Men, dette ser mistenkelig ut som elastisk støt mellom to legemer, der A er i bevegelse og B er i ro før støtet. Da bevares kinetisk energi, i tillegg til bevegelsesmengde.

Da kan du sette opp en ligning for bevegelsesmengde, i tillegg til bevegelsesenergi:

 

chart?cht=tx&chl=p_{A,0} + p_{B,0} = p_{A,1} + p_{B,1}

 

chart?cht=tx&chl=m_A v_{A,0} + m_B v_{B,0} = m_A v_{A,1} + m_B v_{B,1}

 

Hvis så chart?cht=tx&chl=m_A = m og

chart?cht=tx&chl=m_B = 3 m_A = 3m, samt

chart?cht=tx&chl=v_{B,0} = 0 forenkles dette til:

 

chart?cht=tx&chl=v_{A,0} = v_{A,1} + 3 v_{B,1}

  • Liker 1
Lenke til kommentar

Det går ikke å komme direkte fra (3) til (4)

Det du kan skrive fra (3) er:

chart?cht=tx&chl=u_A = \sqrt{v_A^2 + 3 v_B^2}

 

Men, dette ser mistenkelig ut som elastisk støt mellom to legemer, der A er i bevegelse og B er i ro før støtet. Da bevares kinetisk energi, i tillegg til bevegelsesmengde.

Da kan du sette opp en ligning for bevegelsesmengde, i tillegg til bevegelsesenergi:

 

chart?cht=tx&chl=p_{A,0} + p_{B,0} = p_{A,1} + p_{B,1}

 

chart?cht=tx&chl=m_A v_{A,0} + m_B v_{B,0} = m_A v_{A,1} + m_B v_{B,1}

 

Hvis så chart?cht=tx&chl=m_A = m og

chart?cht=tx&chl=m_B = 3 m_A = 3m, samt

chart?cht=tx&chl=v_{B,0} = 0 forenkles dette til:

 

chart?cht=tx&chl=v_{A,0} = v_{A,1} + 3 v_{B,1}

 

Har satt Va1 og Vb1 er start fart i tillegg Va2 og Vb2 er sluttfart

Mb= 3Ma

 

Ut i fra bevaring av kinetisk energi regnet jeg fram til:

post-435273-0-11863800-1547911580_thumb.jpg

 

Og skulle sette Va2 inn i bevaring av bevegelsesmengde likning

post-435273-0-94460700-1547911746_thumb.jpg

 

Da havner jeg bare opp med

post-435273-0-96626100-1547912052_thumb.jpg

 

Hmm

 

 

Dette er oppgaven forresten

post-435273-0-26091100-1547912689_thumb.jpg

Endret av Steeffen
Lenke til kommentar
Gjest Slettet+45613274

Noen som kan forklare dette? Skal gjennomgå trigonometri-kapitlet, men det er lenge til. Men må altså løse denne oppgaven allerede av en eller annen grunn.

 

attachicon.giftrapes.PNG

Jeg regner med selve utregningen går fint?

 

Jeg ser to umiddelbare triks. Kvadratroten av 169 er 13, og 40-13=27. Om vi slenger på pythagoras, så tipper jeg du ser løsningen :)

 

Edit: om det er vanskelig å se så kan det bli lettere av å dele opp figuren i mindre og enklere figurer (trekanter og firkanter)

Endret av Slettet+45613274
Lenke til kommentar

Jeg regner med selve utregningen går fint?

 

Jeg ser to umiddelbare triks. Kvadratroten av 169 er 13, og 40-13=27. Om vi slenger på pythagoras, så tipper jeg du ser løsningen :)

 

Edit: om det er vanskelig å se så kan det bli lettere av å dele opp figuren i mindre og enklere figurer (trekanter og firkanter)

Takk for rask respons! Utregningen er grei, var bare det med å vise at det stemmer. Men nå tror jeg at det skal gå greit :)

Lenke til kommentar

Har satt Va1 og Vb1 er start fart i tillegg Va2 og Vb2 er sluttfart

Mb= 3Ma

 

Ut i fra bevaring av kinetisk energi regnet jeg fram til:

attachicon.gift1.JPG

 

Og skulle sette Va2 inn i bevaring av bevegelsesmengde likning

attachicon.gift2.JPG

 

Da havner jeg bare opp med

attachicon.gift4.JPG

 

Hmm

 

 

Dette er oppgaven forresten

attachicon.gift1.JPG

Tror jeg var litt utydelig her.

Du skriver først

chart?cht=tx&chl=V_{A,1}^2 = V_{A,2}^2 + 3V_{B,2}^2

Løser du dette uttrykket for chart?cht=tx&chl=V_{A,1} får du:

chart?cht=tx&chl=V_{A,1} = \sqrt{V_{A,2}^2 + 3V_{B,2}^2}

 

Kvadratroten av summen av to tall er ikke det samme som summen av kvadratroten av de samme tallene:

chart?cht=tx&chl=\sqrt{V_{A,2}^2 + V_{B,2}^2}   chart?cht=tx&chl=\neq   chart?cht=tx&chl=\sqrt{ V_{A,2}^2} + \sqrt{V_{B,2}^2}

Hvis du ønsker en rask måte å sjekke dette på, kan jeg foreslå å sette inn tallene 3 og 4:

chart?cht=tx&chl=\sqrt{3^2 + 4^2} = \sqrt{25} = 5

chart?cht=tx&chl=\sqrt{3^2} + \sqrt{4^2} = 3 + 4 = 7

Regner med du ser forskjellen  :)

 

 

Ut fra at startfarten til den lette klossen er s

 

Det du ender opp med er to ligninger med to ukjente:

(1) s^2 = V_{A,2}^2 + 3V_{B,2}^2

(2) s= V_{A,2} + 3V_{B,2}

 

Den ene ligningen (1) kommer fra bevaring av kinetisk energi.

Den andre ligningen (2) kommer fra bevareing av bevegelsesmengde.

Lenke til kommentar

Tror jeg var litt utydelig her.

Du skriver først

chart?cht=tx&chl=V_{A,1}^2 = V_{A,2}^2 + 3V_{B,2}^2

Løser du dette uttrykket for chart?cht=tx&chl=V_{A,1} får du:

chart?cht=tx&chl=V_{A,1} = \sqrt{V_{A,2}^2 + 3V_{B,2}^2}

 

Kvadratroten av summen av to tall er ikke det samme som summen av kvadratroten av de samme tallene:

chart?cht=tx&chl=\sqrt{V_{A,2}^2 + V_{B,2}^2}   chart?cht=tx&chl=\neq   chart?cht=tx&chl=\sqrt{ V_{A,2}^2} + \sqrt{V_{B,2}^2}

Hvis du ønsker en rask måte å sjekke dette på, kan jeg foreslå å sette inn tallene 3 og 4:

chart?cht=tx&chl=\sqrt{3^2 + 4^2} = \sqrt{25} = 5

chart?cht=tx&chl=\sqrt{3^2} + \sqrt{4^2} = 3 + 4 = 7

Regner med du ser forskjellen  :)

 

 

Ut fra at startfarten til den lette klossen er s

 

Det du ender opp med er to ligninger med to ukjente:

(1) s^2 = V_{A,2}^2 + 3V_{B,2}^2

(2) s= V_{A,2} + 3V_{B,2}

 

Den ene ligningen (1) kommer fra bevaring av kinetisk energi.

Den andre ligningen (2) kommer fra bevareing av bevegelsesmengde.

 

 

post-435273-0-42235100-1547994434_thumb.jpg

 

Ser ikke hva jeg skal sette inn hvor.. :s

Lenke til kommentar

For den første oppgaven:

Tegn opp punktene du vet eksisterer på et papir først, så setter du d sånn ca der du forventer at D skal være. Da blir det lettere å jobbe videre.

Anta så at punktet D ligger i (X,Y), regn ut de fire forskjellige vektorene nevnt i oppgaven

 

Hva er definisjonen for en parallell vektor? Hva er chart?cht=tx&chl=\vec{BC} ?

 

Hvis chart?cht=tx&chl=\vec{CD} står normalt på chart?cht=tx&chl=\vec{AC}, hva får du ved å regne ut chart?cht=tx&chl=\vec{CD} \cdot \vec{AC} ?

 

 

For den andre oppgaven:

Mener du chart?cht=tx&chl=f(x)=e^4-x^2

Eller chart?cht=tx&chl=f(x)=e^{4-x^2} ?

 

Først og fremst: tegn figur!

Det er en likesidet trekant, så flere løsninger eksisterer. Mitt forslag er å dele trekanten i to på y-aksen, og så regne ut arealet derfra.

  • Liker 1
Lenke til kommentar

Dette er ikke en oppgave men noe jeg lurer på fremgangsmåten på. 

Si at du har en vinkel på f.eks 55,2 grader som består av vektorene ab som har en lengde på 5cm langs x-aksen og ac vektor med en lengde på 6,5cm i en vinkel på 55,2 grader. Er det en måte å finne ut lengden av bc uten å tegne det inn i et koordinatskjema for så å gå om ba og bc vektor. For ved å bruke den metoden så vil det bli unøyaktig. Er det en måte å regne det ut på? Tenkte litt på b-c ved 0 grader delt på cos vinkel a, men det ville krevd en eksponentialfunksjon regner jeg med.  

Lenke til kommentar

Dette er ikke en oppgave men noe jeg lurer på fremgangsmåten på. 

Si at du har en vinkel på f.eks 55,2 grader som består av vektorene ab som har en lengde på 5cm langs x-aksen og ac vektor med en lengde på 6,5cm i en vinkel på 55,2 grader. Er det en måte å finne ut lengden av bc uten å tegne det inn i et koordinatskjema for så å gå om ba og bc vektor. For ved å bruke den metoden så vil det bli unøyaktig. Er det en måte å regne det ut på? Tenkte litt på b-c ved 0 grader delt på cos vinkel a, men det ville krevd en eksponentialfunksjon regner jeg med.  

Ja, du kan bruke cosinussetningen til dette. Siden du ønsker å finne er da de to andre sidene kvadrert og lagt sammen, trukket fra to av dem ganget sammen i tillegg ganget med cosinus til den motstående vinkelen. Mer presist:

 

chart?cht=tx&chl=BC^2 = AB^2 + AC^2 - 2 \cdot AB \cdot AC \cdot cos(a)

 

:)

Endret av -sebastian-
  • Liker 1
Lenke til kommentar
Gjest Slettet+45613274

Dette er ikke en oppgave men noe jeg lurer på fremgangsmåten på.

Si at du har en vinkel på f.eks 55,2 grader som består av vektorene ab som har en lengde på 5cm langs x-aksen og ac vektor med en lengde på 6,5cm i en vinkel på 55,2 grader. Er det en måte å finne ut lengden av bc uten å tegne det inn i et koordinatskjema for så å gå om ba og bc vektor. For ved å bruke den metoden så vil det bli unøyaktig. Er det en måte å regne det ut på? Tenkte litt på b-c ved 0 grader delt på cos vinkel a, men det ville krevd en eksponentialfunksjon regner jeg med.

Jepp, det heter cosinus-setningen, som kan forstås å være en generalisering av pythagoras.

 

https://en.m.wikipedia.org/wiki/Law_of_cosines

 

Edit: epsilon too late.

Endret av Slettet+45613274
Lenke til kommentar

Ja, du kan bruke cosinussetningen til dette. Siden du ønsker å finne er da de to andre sidene kvadrert og lagt sammen, trukket fra to av dem ganget sammen i tillegg ganget med cosinus til den motstående vinkelen. Mer presist:

 

chart?cht=tx&chl=BC^2 = AB^2 + AC^2 - 2 \cdot AB \cdot AC \cdot cos(a)

 

:)

Takk sebastian og failern, den hadde jeg helt glemt. Tenkte ikke på før nå at jeg kunne dele den i to rettvinklede trekanter heller. 

Endret av wertyuiopå
Lenke til kommentar

Takk sebastian og failern, den hadde jeg helt glemt. Tenkte ikke på før nå at jeg kunne dele den i to rettvinklede trekanter heller. 

Du kunne også forsøkt å dekomponere to vektorer, og så regne deg frem til chart?cht=tx&chl=\vec{BC} derfra

 

Setter du chart?cht=tx&chl=|\vec{A}| = A, chart?cht=tx&chl=|\vec{B}| = B, og vinkelen mellom er chart?cht=tx&chl=\theta

Og så sier at chart?cht=tx&chl=\vec{A} går parallell med x-aksen, slik at chart?cht=tx&chl=\vec{B} = [b \cos{\theta}, B \sin{\theta}]

 

Da blir det blir en smal sak å beregne chart?cht=tx&chl=|\vec{C}|, som blir den siste siden i trekanten

(Du kommer nok fort frem til akkurat samme svar som nevnt over :wee: )

Lenke til kommentar

Jeg trenger å få bekreftet noe knyttet til rekker:

 

Gitt p-rekken (1/n^p) og den harmoniske rekken (1/n), gjelder reglene også da for hvilke som helst konstant i telleren?

Eks: Vil (100/n) == (1/n)? Jeg går ut fra at det er tilfellet, siden den harmoniske rekken konvergerer og da vil den vel også konvergere ved hvilken som helst konstant k i telleren?

Lenke til kommentar
Gjest Slettet+45613274

Jeg trenger å få bekreftet noe knyttet til rekker:

 

Gitt p-rekken (1/n^p) og den harmoniske rekken (1/n), gjelder reglene også da for hvilke som helst konstant i telleren?

Eks: Vil (100/n) == (1/n)? Jeg går ut fra at det er tilfellet, siden den harmoniske rekken konvergerer og da vil den vel også konvergere ved hvilken som helst konstant k i telleren?

Jeg tror du blander rekker og følger. Den harmoniske rekken divergerer. Følgen 1/n konvergerer til 0 når n går mot uendelig.

 

p-rekken 1/n^p konvergerer for alle reelle p>1. Når det gjelder spørsmålet ditt så er det korrekt tenkt (bortsett fra at den harmoniske rekken faktisk ikke konvergerer). Sum (c/n^p) konvergerer for alle c reelle og reelle p>1.

Endret av Slettet+45613274
Lenke til kommentar

Opprett en konto eller logg inn for å kommentere

Du må være et medlem for å kunne skrive en kommentar

Opprett konto

Det er enkelt å melde seg inn for å starte en ny konto!

Start en konto

Logg inn

Har du allerede en konto? Logg inn her.

Logg inn nå
×
×
  • Opprett ny...